LSAT and Law School Admissions Forum

Get expert LSAT preparation and law school admissions advice from PowerScore Test Preparation.

User avatar
 Dave Killoran
PowerScore Staff
  • PowerScore Staff
  • Posts: 5853
  • Joined: Mar 25, 2011
|
#41296
Complete Question Explanation
(The complete setup for this game can be found here: lsat/viewtopic.php?t=13756)

The correct answer choice is (C)

This is a maximum question, wherein you are asked to separate two variables by the maximum number of spaces. Interestingly, H and S are the two randoms in this game, and so to schedule them as far apart as possible would be to place them in the first and last slots, assuming that the two rule formations in play could fit in slots 2-6. As we know from the diagram in question #1, the two rules can occupy slots 2-6, and thus we can place H and S in the first and last slots in some order:
F93_Game_#1_#3_diagram 1.png
Consequently, answer choice (C) is correct.
You do not have the required permissions to view the files attached to this post.
User avatar
 Arelyramirezdiaz
  • Posts: 2
  • Joined: Feb 08, 2021
|
#83936
Hi for this case I had a similar answer to yours except instead of just G for spot #2 I listed it as G/P and for spot #5 I listed it as P/G. Did I not consider something? I got the answer right, but I did not feel confident because of the G/P situation.
 Jeremy Press
PowerScore Staff
  • PowerScore Staff
  • Posts: 1000
  • Joined: Jun 12, 2017
|
#83947
Hi Arely,

Your diagram that shows G/P as possibilities for spaces 2 and 5 is correct, so nice job! The crucial first step for this question is to see that, to separate H and S by as many spaces as possible, one of them has to be on space 1 and one has to be on space 7. This immediately narrows the possible answer choices to C and D (none of the others lists H or S in space 1). Between C and D, we have to choose C, because the difficulty of placing that LO block, with N coming after it, will force us to put either G or P on space 2 (and the other one on space 5). To validate answer choice C, you can diagram as the original explanation post has diagrammed, to show that it is possible to have H on 1, G on 2, and L on 3. But you're right that we can't tell from the given condition whether G or P will end up on spot 2. That doesn't matter, because the question asks about what could be true. In other words, it's only asking you to find one of the possible solutions: the one depicted in the diagram where G is on 2.

I hope this helps!

Get the most out of your LSAT Prep Plus subscription.

Analyze and track your performance with our Testing and Analytics Package.